You are on page 1of 69

GEC 3

Mathematics in the
Modern World

Course
Modules

Weeks 1 – 6
These modules were prepared for use of Isabela State University Faculty
and students in GEC 3: Mathematics in the Modern World for the First
Semester of School Year 2020-2021.
Topics in the modules are suggested to be covered within the first six
weeks of the semester.

Supervision:
Helena B. Florendo
Dean
College of Arts and Sciences

Nanette D. Sayo
ARA Director
ISU Echague Campus

Content and Review:


Marion Faye Q. Ariola – ISU Echague
Januard Dagdag – ISU San Mariano
Rowena M. Del Rosario – ISU Echague
Rosario E. Fajardo – ISU Echague
Merridy V. Macutay – ISU Echague
Marianne Jane Antoinette D. Pua – ISU Echague
Ma. Lailano B. Walo – ISU Echague

We also acknowledge feedback and response from the following Faculty


members:

Lorraine Abe – ISU Cauayan


Gilbert Eustaquio – ISU Santiago Extension
Eugelyn Felix – ISU Angadanan
Grace D. Ladia – ISU Angadanan
Marilene Matusalem – ISU Roxas
Alberto Sia Jr. – ISU Cabagan
Myla Velasco – ISU San Mateo
GEC 3 Mathematics in the Modern World

Description:

This course deals with nature of mathematics, appreciation of its


practical, intellectual, and aesthetic dimensions, and applications of
mathematical tools in daily life.

The first set of modules is designed to cover the first section of the course
which is an introduction to the nature of mathematics as an exploration of
patterns, as a powerful language, and as an application of inductive and
deductive reasoning. By exploring these topics, students are encouraged to
go beyond the typical understanding of mathematics as merely a set of
formulas but as a source of aesthetics in patterns of nature, for example, and
a rich language in itself governed by logic and reasoning.
Module 1
Mathematics in Our World
MODULE 1
The Nature of Mathematics: Mathematics in Our World

1.1 Introduction
Have you ever asked yourself where all mathematics come from? Is it
invented or discovered?
We cannot deny the fact that these days, more mathematics become part
of our daily lives. We feel the need to be cautious of our ways because we
hear news about increasing Covid-19 cases in our country and even in our
locality.
Numbers that are presented to us daily become so powerful that they are
used as bases for decisions and actions of our leaders to prevent Covid-19
cases from increasing.
What we gave as an example is just one of the many mathematics that
we encounter daily. During this very challenging time, we believe that
everyone would value mathematics and would want to know more about it.
For the first section, we will learn about mathematics as a useful way of
thinking about nature and the world. We hope that after this part, you will
understand mathematics as not merely a set of formulas but as a source of
relevant information that can help us in dealing with our daily lives.
1.2 Learning Outcome
After finishing this module, you are expected to
a. identify patterns in nature and regularities in the world,
b. articulate the importance of mathematics in your life,
c. argue about the nature of mathematics, what it is, how it is
expressed, represented and used, and
d. express appreciation for mathematics as a human endeavor.
1.3 What You Need to Know
Mathematics is a broad system of study. Not one mathematician can
define what mathematics is. If you happen to see the picture of an elephant
and the six blind men, the picture somewhat describes what mathematics is.
In the picture, we view the elephant as mathematics and the six men
represent the mathematicians.

Page 1 of 12
Figure 1.1
Six blind men and an elephant

Each of the six blind men did a good job in describing what they have
sensed. We cannot say that they incorrectly described each part since what
they touched was only part of the whole. The same can be observed in
mathematics. The mathematics that can be described by one mathematician
is just a small part of the whole thing.

1.3.1 Getting to Know Mathematics

1.3.1.1 What is Mathematics?

The simplest mathematical objects are numbers, and the simplest of


nature's patterns are numerical. The phases of the moon make a complete
cycle from new moon to full moon and back again every twenty-eight days.
The year is roughly three hundred and sixty-five days long. People have two
legs, cats have four, insects have six, and spiders have eight. Starfish have
five arms (or ten, eleven, even seventeen, depending on the species).

A very curious pattern indeed occurs in the petals of flowers. In nearly


all flowers, the number of petals is one of the numbers that occur in the
strange sequence 3, 5, 8, 13, 21, 34, 55, 89. For instance, lilies have three
petals, buttercups have five, many delphiniums have eight, marigolds have
thirteen, asters have twenty-one, and most daisies have thirty-four, fifty-five,
or eighty-nine. You don't find any other numbers anything like as often. There
is a definite pattern to those numbers: each number is obtained by adding
the previous two numbers together. For example, 3 + 5 = 8, 5 + 8 = 13, and
so on. Some examples are given in Figure 1.2.

Page 2 of 12
Figure 1.2
Different flowers with corresponding petal count

The same numbers can be found in the spiral patterns of seeds in the
head of a sunflower. This particular pattern was noticed many centuries ago
and has been widely studied ever since, but a really satisfactory explanation
was not given until 1993.

A sunflower seed illustrates that the number of clockwise spirals is 34


and the number of counterclockwise spirals is 21, where 21 and 34 are two
consecutive numbers in the given sequence.

Figure 1.3
Spiral patterns of sunflower seed arrangement

Pinecones also illustrate the number of clockwise and counterclockwise


spirals. Figure 1.4 shows the numbers 8 and 13.

Page 3 of 12
Figure 1.4
Spiral pattern in a pinecone

With these few examples, we must understand that human mind and
culture had long developed a formal system for classifying, recognizing, and
exploiting patterns. We call it mathematics. Patterns observed paved the way
for the origin of counting, the discovery and creation of geometric patterns,
wave patterns in water and on land, patterns of movement, and fractals: the
new science of irregularity; a never-ending pattern.
Figure 1.5 shows other patterns and regularities in nature.

Figure 1.5
Different patterns in nature
One of the most frequently occurring patterns in nature is the Fibonacci
sequence. It is a sequence of numbers starting with 1, then followed by 1,
and then followed by the sum of 1 and 1 which is 2, followed by the sum of 1
and 2, which 3 and so on. The sequence is given by

1, 1, 2, 3, 5, 8, 13, 21, …

Page 4 of 12
Leonardo of Pisa, also known as Fibonacci (c. 1170–1250), is one of the
best-known mathematicians of medieval Europe. In 1202, after a trip that
took him to several Arab and Eastern countries, Fibonacci wrote the book
Liber Abaci. This book contains a problem created by Fibonacci that concerns
the birth rate of rabbits. Here is a statement of Fibonacci’s rabbit problem.

At the beginning of a month, you are given a pair of newborn


rabbits. After a month the rabbits have produced no offspring;
however, every month thereafter, the pair of rabbits produces
another pair of rabbits. The offspring reproduce in exactly the
same manner. If none of the rabbits dies, how many pairs of
rabbits will there be at the start of each succeeding month?

The solution of this problem is a sequence of numbers that we now call


the Fibonacci sequence.

Figure 1.6
Rabbit reproduction modeled by the Fibonacci sequence

The same sequence models what we previously discussed on petal


counts and pinecones and sunflower.
1.3.1.2 Where is mathematics?
Try to look around you. You will see that nature is filled with patterns.
Thus, mathematics works around us and we see it in clues and hints in
nature. In our daily routine, we follow a pattern of the things we normally do.
In people, communities, and events, we can observe many different patterns.
We are used to some patterns around us that if that pattern is changed,
things may be strange or exceptional. If somebody walks at a rhythm of left-

Page 5 of 12
left-right-right, we consider it strange to move that way since we are used to
walking with a left-right-left-right pattern. If a tree bears an exceptionally
large fruit, they may treat this as a special phenomenon. Here, we see that
there is deep-seated belief that exceptions to patterns are special.
During this pandemic, we feel strange because some of the patterns we
used to observe have changed. Still, we need to get used to the new and arising
pattern brought about by changes in nature and our surroundings.
1.3.1.3 What is mathematics for?
Nature is full of patterns, most of which are things of beauty. Thus, we
cannot help but sit back and admire them. However, each nature’s pattern is
believed to be a puzzle that needs to be analyzed.
Mathematics makes it possible to solve these puzzles because of its
systematic way of digging out the rules and structures that lie behind these
puzzles of nature. It has a way of organizing patterns, regularities, and
irregularities. It is able to predict or even control weather, epidemics and it
provides tools for calculations. Because of mathematics’ ability to exploit
patterns, it can even provide more questions to think about.
1.3.1.4 What is mathematics about?
When we hear the word mathematics, we usually relate it to numbers. It
is true because numbers are the heart of mathematics. But this is just a
small part of mathematics. We will discuss some important aspects of
mathematics. We start off discussing numbers.
1.3.1.4a Numbers
The simplest numbers are those used in counting. These numbers were
discovered long before the symbols 1, 2, 3, … are used. Back then, they did
not use these symbols for numbers. They used their fingers, twigs, stones,
and objects that can help them count. At present the set of counting numbers
is also called the set of natural numbers.
Between 400 and 1200 AD, the concept of zero was invented and
accepted as denoting a number. History books say that the key idea was the
invention of a symbol for “nothing”.
The next extension of the number concept is the invention of the negative
numbers. A negative number may be thought to represent debt. Many other
interpretations may be used. For example, a negative temperature in Celsius
scale indicate a temperature lower than freezing point. An object with
negative velocity is moving backward. This means that the same
mathematical object can represent more than one aspect of nature.
If the counting numbers, zero, and the negative counting numbers are
combined, a new set of numbers is formed. This is the set of integers.

Page 6 of 12
These numbers are not enough; we need fractions especially for most
commercial transactions. Positive and negative fractions together with the
integers are called rational numbers.
Numbers that cannot be expressed as fractions having a unique property
of non-terminating and non-repeating decimals are given the name
“irrational”. An example of an irrational number is the square root of 2,
denoted as √2. If we use the calculator, we will obtain √2 =1.4142135624…,
a decimal which is non-terminating and non-repeating.
Rational numbers and irrational numbers combined form a larger
number set called the set of real numbers.
The introduction of square roots of negative numbers led to the invention
of the imaginary and complex numbers.
So now we have five number systems, each more inclusive that the
previous: natural numbers, integers, rationals, real numbers, and complex
numbers.
1.3.1.4b Operations
Mathematics is not only about numbers. We are familiar with addition,
subtraction, multiplication, and division. These are called operations. In
general, an operation is something we apply to two or more mathematical
objects to get another mathematical object. So if you add, say, 2 and 3, you
will get a third number, which is 5.
1.3.1.4c Function
If you start with a number and form its double, you get another number.
So if you start with 5, you will get 10. The term for such an “object” is
function. You can think of a function as a mathematical rule that starts
with a mathematical object – usually a number – and associates to it another
object in a specific manner. In the given example, the association is the rule
of doubling the number.
Functions are often defined using algebraic formulas but they may also
be defined in other convenient ways. Another term with the same meaning
as “function” is transformation: the rule transforms the first object into the
second.
1.3.1.4d Thingification of Processes
The dictionary term for “thingification” is reification. However, in the
discussion the term “thingification” is chosen for emphasis.
Mathematical “things” have no existence in the real world: they are
abstractions. But mathematical processes are also abstractions, so processes
are no less “things” than the “things” to which they are applied. The number
“2”, for example, is not actually a thing but a process – the process you carry

Page 7 of 12
out when you associate two cats with the symbols “1, 2” recited in turn. A
number is a process that has long ago been thingified so thoroughly that
everybody thinks of it as a thing.
Mathematics is not just a collection of isolated facts: it is more like a
landscape; a geography where users get to travel through what seem to be an
impermeable forest. There is a metaphorical feeling of distance. For instance,
the fact that the circumference of a circle is 1𝜋 (pi) times its diameter is very
close to the fact that the circumference of a circle is 2𝜋 times its radius. The
connection between these two facts is immediate: the diameter is twice the
radius.
Unrelated ideas are more distant from each other. For example, the fact
that there are exactly six different ways to arrange three letters of the alphabet
is distant from facts about circles.
1.3.1.4e Proof
The link between ideas in mathematics is made possible by proof. Proof
determines the route from one fact to another. Textbooks of mathematical
logic say that a proof is a sequence of statements, each of which either follows
from previous statements in the sequence or from agreed axioms unproved
but explicitly stated assumptions that in effect define the area of mathematics
being studied. A proof can be equated to a novel which must tell an interesting
story, rather than just a sequence of sentences.
A mathematical proof is a story about mathematics that works. The story
must not have gaps, and it certainly must not have an unbelievable plot line.
The rules are stringent: in mathematics, a single flaw is fatal. Moreover, a
subtle flaw can be just as fatal as an obvious one.
1.3.1.5 How is mathematics done?
Mathematics is done out of curiosity, with penchant for seeking patterns
and generalities, with a desire to know the truth, with trial and error, without
fear of facing more questions and problems to solve.
1.3.1.6 Who uses mathematics?
Practically, everyone uses mathematics.
The Mathematicians: pure and applied use mathematics to further
explore undiscovered concepts and information in this field. Dr. Guido David
and his team use mathematics to predict the number of Covid-19 cases for
the succeeding months. This was used by the government to decide on actions
to be done.
Scientists: natural and social uses mathematics to describe a certain
phenomenon and to decide on the results of their research. These are just
few examples of people who use mathematics. All of us has a particular use
of mathematics in our lives.

Page 8 of 12
Thus, different people use different mathematics at different times, for
different purposes, using different tools, with different attitudes.

Learning Activity 1 Patterns in Nature

We can use the Fibonacci numbers to create this spiral that is so common in nature.
Read the directions to help you draw squares on the graph paper. If you follow each step
carefully, you will make a Fibonacci spiral!

1. Look at the graph paper. The first number in the Fibonacci sequence, 1, has
been drawn for you.
2. Go to the square to the right of 1. Outline that little square to represent the
next number in the pattern, another 1.
3. Use the line above the two 1 squares to outline a square that is 2 little squares
long and 2 little squares high. This represents the next number in the sequence
which is 2.
4. Now move to the right of the squares 1 and 2. Use the right side of the 2 squares
and the right side of the second 1 square to draw a square that is 3 little squares
high and 2 little squares long. 3 is the next number I Fibonacci’s pattern.
5. Use the bottom of both 1 squares and the bottom of the 3 square to make the
next number in the pattern – a big square that is 5 little squares long and five
little squares high.
6. Move to the left of the 2 square, the 1 square, and the 5 square. Use their left
edges to make the 8 square.
7. Finally use the top of the 8 square long with the top of the 2 and 3 square to
make a 13 square.

Here is your graph paper:

Page 9 of 12
Learning Activity 1 Patterns in Nature continued

If you followed directions, you will have used the entire graph paper to
make Fibonacci’s Rectangle, also known as the Golden Rectangle. The ratio of
length to that of the width of the golden rectangle is equal to the Golden Ratio,
φ, which is approximately equal to 1.618. The golden ratio is also known as the
divine proportion.

The next step is to draw Fibonacci’s spiral. You just have to connect one
corner of each square with the opposite corner of that square with a sweeping
curve. You may need to practice a few times to get it right.

Compare what you have made to patterns in nature. Try to spot this spiral
in your surroundings. Make a list of animals, plants, and man-made objects
that have this spiral.

Learning Activity 2 Patterns in Nature

Materials:

1 Short bond paper


One-peso coins

Procedure:

1. Using coins of the same size, try to cover as much area of a piece of paper
with coins.
2. Arrange the coins in a square formation.
3. Count and record the number of coins you can place to cover the area of
the bond paper.
4. Arrange the coins in hexagonal formation.
5. Record the number of coins you can place.
6. Compare the results in Step 3 and Step 5.

Which formation is more compact? Justify your answer.

Page 10 of 12
Recommended learning materials and resources
 Arithmetic and Geometric Sequences
https://www.youtube.com/watch?v=MSAWzIlPlkg

 Nature by Numbers
https://www.youtube.com/watch?v=kkGeOWYOFoA

 Mathematics in Nature
https://www.youtube.com/watch?v=Ig9RUaJe00c

 Fibonacci Sequence: Nature’s Code


https://www.youtube.com/watch?v=wTlw7fNcO-0

 Golden Ratio
https://www.youtube.com/watch?v=c8ccsE_IumM

 The Great Math Mystery


https://www.youtube.com/watch?v=mpcpzXuzdQk

Flexible Teaching Learning Modality (FTLM)


Remote (asynchronous)
 module, exercises, problems sets, powerpoint lessons

Assessment Task
Before you answer the following questions, make sure to watch the
Youtube videos given below:
 Nature by Numbers
https://www.youtube.com/watch?v=kkGeOWYOFoA

 The Great Math Mystery


https://www.youtube.com/watch?v=mpcpzXuzdQk
A. Short-response/Essay writing:
1) What new ideas about mathematics did you learn?
2) What is it about mathematics that might have changed your thoughts
about it?
3) What is most useful about mathematics for humankind?

Page 11 of 12
B. Write a 2- to 3-page synthesis focusing on one of the following aspects of
mathematics:
1) Mathematics helps organize patterns and regularities in the world.
2) Mathematics helps predict the behavior of nature and phenomena in
the world.
3) Mathematics helps control nature and occurrences in the world for our
own ends.
Your answers will be graded according to the given standards/basis for
grading:
Score Criteria
Unable to elicit the ideas and concepts from the learning activity, material,
0
or video
Able to elicit the ideas and concepts from the learning activity, material, or
1
video but shows erroneous understanding
Able to elicit the ideas and concepts from the learning activity, material, or
2
video and shows correct understanding
Able to elicit the correct ideas from the learning activity, material, or video
3 and also shows evidence of internalization and consistently contributes
additional thought to the core idea

References
Books:
Aufmann, et. al. 2013. Mathematical Excursions. Third Edition. Cengage
Learning, USA
Stewart, Ian. 1995 The Unreal Reality of Mathematics Nature’s Numbers.
BasicBooks New York
Internet sources:
The Great Math Mystery
https://www.youtube.com/watch?v=mpcpzXuzdQk

Page 12 of 12
Module 2
Mathematical Language
and
Symbols
MODULE 2
The Nature of Mathematics: Mathematical Language and Symbols

2.1 Introduction
The nature of mathematics has become clear to us when we learned in
the first part how it is used to understand nature and our surroundings
because of its ability to explain the different patterns in nature.

Another aspect of mathematics that needs to be studied is that it has its


own language which makes possible many different processes in different
fields of learning.

This module will discuss the characteristics of the mathematics


language, the conventions, basic concepts, and applications. Different
symbols are introduced as language characteristics and some mathematical
concepts are discussed.

2.2 Learning Outcome


After finishing this module, you are expected to:
1. discuss the language, symbols, and conventions of mathematics;
2. explain the nature of mathematics as a language;
3. translate expressions and sentences to mathematical symbols; and
4. acknowledge that mathematics is a useful language

2.3 What You Need to Know


Mathematics is a language that is understood throughout the world. It
is effective in terms of expressing ideas and in communicating ideas to others
without the burden of translating such to different languages.

2.3.1 Characteristics of the Language of Mathematics

Mathematics as a language is precise, concise, and powerful.

It is precise in the sense that it is able to make very fine distinctions


based on definitions. We know that a square is different from a circle because
they were defined differently. Visual representations also tell us that two
mathematical objects are different. We all know that a square has a different
visual representation with a circle. Do you know the difference between a
square and a rectangle? Are all squares rectangles or are all rectangles
squares?

Page 1 of 31
The language of mathematics is concise. It is able to say things briefly
because it has in it different symbols. For example, the sentence “Two plus
three is equal to five” may be expressed concisely as 2 + 3 = 5.

Lastly, this language is powerful. It is able to express complex thoughts


with relative ease.

The language of mathematics can be learned but it requires an effort


similar to learning a foreign language.

2.3.2 The English Language and The Mathematical Language

There are certain similarities between the English language and


mathematical language. A noun in the English language corresponds to a
mathematical expression while a sentence in the English language
corresponds to a mathematical sentence.

In English, nouns are used to name things we want to talk about (like
people, places, and things); whereas sentences are used to state complete
thoughts. A typical English sentence has at least one noun, and at least one
verb.
A mathematical expression is a correct arrangement of mathematical
symbols to represent the object of interest, does not contain a complete
thought, and does not possess a truth value (one will not know if it is true or
false).
The mathematical analogue of a sentence is also called a sentence. A
mathematical sentence must state a complete thought. The table below
shows the analogy.
ENGLISH MATHEMATICS
Name given to an NOUN (person, place, thing) EXPRESSION
object of interest: 1
Examples: Mario, Isabela, book Examples: 2, 3 + 5, 3

A complete SENTENCE SENTENCE


thought: Examples: Examples:
The capital of the province of
Isabela is Ilagan City. 3+5 = 8
The capital of the province of 3 + 5 = 13
Isabela is Cauayan City.

Frequently, we need to work with numbers which are most common types
of mathematical expressions. Even a single number can have numerous
names. For example, the expressions
6 4+2 12 ÷ 2 (4 − 1) + 3 1+1+1+1+1+1

Page 2 of 31
all look different, but are all just different names for the same number.
Just like English sentences, mathematical sentences also have verbs. In
the mathematical sentence ‘4 + 2 = 6’, the verb is ‘ = ’. If you read the sentence
as ‘four plus two equals six’, then it is easy to identify the verb. The equal sign
is one of the most popular mathematical verbs.
Learning Activity 1

Directions. If possible, classify the entries in the list below as:

 an English noun (EN), or a mathematical expression (ME)


 an English sentence (ES), or a mathematical sentence
(MS)

Write your answer in the blanks before looking at the solutions. In


each sentence (English or mathematical), circle the verb.

Examples

2𝑥 + 5𝑦 ME
Three is a crowd. ES

Start Here:

1. cat
2. 2
3. The word ‘chat’ begins with the letter ‘t’.
4. 5 + 2 = 4
5. 7 − 3
6. 5 − 3 = 2
7. The cat is white.
8. 𝑥
9. 𝑥 = 1
10. 𝑥 − 1 = 0
11. 𝑡 + 3
12. 𝑡 + 3 = 3 + 𝑡
13. This sentence is false.
14. 𝑥 + 0 = 𝑥
15. 1 · 𝑥 = 𝑥

2.3.3 Conventions in the Mathematical Language

Just like any language, the mathematical language uses conventions. In


the English language, the use of capital letters for proper nouns is
conventional. Thus, our place Isabela is written with the first letter
capitalized. This convention helps us distinguish between common nouns
and proper nouns. In mathematics, there are different conventions for us to
distinguish between different types of mathematical objects.

Page 3 of 31
Note that sentences state a complete thought, but nouns and expressions
do not. For example, read aloud: 7. What about 7? Now read aloud: 9 − 2 = 7.
This states a complete thought about the number 7.
Sentences can also be true or false. The notion of truth, the property of
being true or false, is of fundamental importance in mathematics.
Mathematical sentences are declarative in nature and we can determine
whether they are true or false.
We have mentioned earlier that the mathematical language is precise. In
order to communicate effectively, we must agree on the meanings of certain
words and phrases because confusion may result from ambiguities. Consider
the following conversation in a car at a noisy intersection:
Mario: “Turn left!”

Dana: “I didn’t hear you. Left?”

Mario: “Right!”

Question: Which way will Dana turn? It depends on how she interprets
the word ‘right’. If she interprets ‘right’ as the opposite of ‘left’, then she will
turn right. If she interprets ‘right’ as ‘correct,’ then she will turn left. The word
‘right’ caused confusion.
There is much less ambiguity allowed in mathematics than in English.
Ambiguity is avoided because mathematics has in it the use of definitions. By
defining words and phrases, it is assured that everyone agrees on their
meaning. Here is our first definition:

DEFINITION 2.1 (expression)

An expression is the mathematical analogue of an English noun; it is a correct


arrangement of mathematical symbols used to represent a mathematical object of
interest. An expression does NOT state a complete thought; in particular, it does
not make sense to ask if an expression is true or false.

Most mathematics books do not define expression. It must be clear that


expressions and sentences are totally different. They do not overlap. If
something is an expression, then it is not a sentence. If something is a
sentence, then it is not an expression.
There are many types of expressions in mathematics. Some of these are
numbers, sets, functions, ordered pairs, matrices, vectors. Most common
types are numbers, sets, and functions.

Page 4 of 31
In mathematics, some expressions can have many different names. The
name depends on what we are doing with the expression. An example is the
number 1 which goes with the following names:

1 1 1 1 1 3 1 1 1
+ 2−1 + + + +
2 2 3 3 3 3 2 4 4

1 1
The name 2 + 2, for example, is appropriate if we need to divide a chocolate
1 1 1
bar evenly for two kids. The name + 3 + 3 is appropriate if we only have a
3
one-third cup measure but needs 1 cup of sugar. Do you know when it is
1 1 1
appropriate to name 1 as 2 + 4 + 4 ?

Learning Activity 2

Direction. Give a name for the number ‘3’ that would be appropriate in each
situation:
Examples:

Three identical rods must be cut 3 3 3 3 3 3


+ + + + +
equally to be distributed to 6 6 6 6 6 6 6
students.

Start Here:

1. three candy bars must be equally


divided among three people
2. three candy bars must be equally
divided among six people
3. you need three cups of flour, but
only have a one-quarter cup
measure
4. you need three cups of flour, but
only have a one-half cup measure

The most common problem involving expressions in mathematics is


simplification. To simplify an expression means to give another name for the
expression in a way that is simpler. When is an expression simpler?

Page 5 of 31
The notion of ‘simpler’ can have different meanings:
An expression is simpler if Example
 it has fewer symbols Both expressions 2 + 3 + 4 and 9 are names for
the same number but 9 uses fewer symbols.
 it has fewer operations Both expressions 3 + 3 + 3 + 3 + 3 and 5 ∙ 3 are
names for the same number but the latter has
fewer operations.
 it is better suited for current use 1 foot
The name 12 inches is a great name for the
number 1 if we need to convert units of inches
to units of feet.
 it follows the preferred style or 2 1
The expressions 4 and 2 are both names for the
format 1
same number. People usually prefer 2 because
it is in the ‘reduced form’ or ‘simplest form’.

We now move to defining precisely what a mathematical sentence means.


The definition follows:

DEFINITION 2.2 (mathematical sentence)

A mathematical sentence is the analogue of an English sentence; it is a correct


arrangement of mathematical symbols that states a complete thought. It makes
sense to ask about the TRUTH of a sentence: Is it true? Is it false? Is it sometimes
true/sometimes false?

The sentence ‘ 1 + 2 = 3 ’ is read as ‘one plus two equals three’. A


complete thought is being stated, which in this case is true.
A question commonly encountered, when presenting the sentence
example ‘ 1 + 2 = 3 ’, is the following: If ‘ = ’ is the verb, then what is the
‘+’? The symbol ‘ + ’ is a connective; a connective is used to ‘connect’ objects
of a given type to get a ‘compound’ object of the same type. Here, the numbers
1 and 2 are ‘connected’ to give the new number 1 + 2.

There are two primary ways to decide whether something is a sentence,


or not:
• Read it aloud, and ask yourself the question: Does it state a complete
thought? If the answer is ‘yes’, then it is a sentence. Notice that
expressions do not state a complete thought. Consider, for example,
the number ‘ 1 + 2 ’ . Say it aloud: ‘one plus two’. Have you stated a
complete thought? NO! But, if you say: ‘ 1 + 2 = 4 ’ , then you have
stated a complete thought.

Page 6 of 31
• Alternately, you can ask yourself the question: Does it make sense to
ask about the TRUTH of this object? Consider again the number ‘ 1 +
2 ’ . Is ‘ 1 + 2 ’ true? Is ‘ 1 + 2 ’ false? These questions do not make
sense, because it does not make sense to ask about the truth of an
expression.
Here are some more examples of mathematical sentences and their truth
values.

Mathematical sentence True, False, Sometime true/sometimes false


𝑥 2 + 2𝑥 = 𝑥(𝑥 + 2) True
𝑥+2< 𝑥−3 False
𝑓 (2) = 3 Sometimes true/sometimes false

The first sentence, 𝑥 2 + 2𝑥 = 𝑥(𝑥 + 2), is true for all real numbers following
the distributive axiom. The second sentence, 𝑥 + 2 < 𝑥 − 3, is false for all real
numbers. No real number will make the sentence true. The last sentence,
𝑓 (2) = 3, is true if we define 𝑓 (𝑥 ) = 𝑥 + 1 so that 𝑓(2) = 2 + 1 = 3. However, if
we define 𝑓 (𝑥 ) = 3𝑥 then 𝑓 (2) = 6 ≠ 3 which makes the given sentence false.
Thus, this sentence is sometimes true/sometimes false.
Learning Activity 3

Direction. Classify the truth of each sentence: always true (T); always
false (F); or sometimes true/sometimes false (ST/SF).

Examples

1+2 =3 T
ST/SF; The
sentence is
𝑥+5=2 true if 𝑥 = −3.
Otherwise, it is
false.
Start Here:

1. 𝑥 ÷ 3 = 2
2. 3 + 5 + 4 = 2 + 7 + 3
3. 𝑥 − 1 = 3
4. 5 − 1 = 5
5. 5 − 1 ≤ 5
6. 5 − 1 < 5
7. 1 + 2 + 𝑥 = 𝑥 + 1 + 2
8. 5𝑥 + 2𝑦 = 7
9. 3 − 5 = 4
10. 3 × 7 + 3 ÷ 3 = 8

Page 7 of 31
2.3.4 Grammar in the Mathematical Language

Grammar is the set of structural rules governing the use of symbols


representing mathematical objects.

Observe that in mathematics, we follow rules in translating from English


language to the mathematical language. Like any translation, this one is
not without problems.

There are certain difficulties in the mathematical language. Here are


some.

1. The word “is” could mean equality, inequality, or membership in a set


depending on how they are used in a sentence.

Consider the sentences below:

1. 5 is the square root of 25.


2. 5 is less than 10.
3. 5 is a prime number.

In the first of these sentences, is could be replaced by equals: it


says that two objects, 5 and the square root of 25, are in fact one and
the same object. This is similar in use to the English sentence Ilagan
City is the capital of Isabela.

In the second sentence, is plays a completely different role. The


words less than 10 form an adjectival phrase, specifying a property that
numbers may or may not have. This is similar in use to the English
sentence Grass is green.

As for the third sentence, the word is there means is an example


of, as it does in the English sentence Isabela is a province. This could
also be a membership property meaning Isabela belongs to the set of
provinces.

These differences are reflected in the fact that the sentences do not
resemble each other when they are written in a more symbolic way.
Thus, we give the corresponding sentences in symbols below.

Sentence Sentence in symbol


5 is the square root of 25 5 = √25
5 is less than 10 5 < 10
5 𝜖 𝑃 where 𝑃 is the set of all
5 is a prime number
prime numbers

Page 8 of 31
The third sentence used the symbol ϵ which indicates membership
to a set (the concept of sets will be discussed in detail in the succeeding
topics). The sentence would normally not be written symbolically
because the concept of a prime number is not quite basic enough to
have universally recognized symbols associated with it. However, it is
sometimes useful to do so which means we need to invent a suitable
symbol. In this case, we may use the letter 𝑃 to denote the set of all
primes.

Learning Activity 4

Direction. Classify the use of “is” in each of the following mathematical


sentences: equality (E), inequality (I), membership (M)

Examples
3
2 is the √8. E
2𝑥 is less than or equal to 5 I
3 is an integer. M
Start Here:

1. 𝑥 divided by 3 is 2.
2. 5 minus 1 is not equal to 5.
3. 1 plus 2 plus 𝑥 is equal to 𝑥 plus 1 plus 2
4. 5 is a natural number.
5. 2 is in the set of all even integers.
6. 5 minus 1 is less than 5.
7. 4 is a multiple of 2.
2
8. 5 is a rational number.
9. 3 plus 5 is less than 10.
10. 3 times 7 plus 3 divided by 3 is 22.

2. A number in a sentence may be of cardinal, ordinal, or nominal type.

Numbers in mathematics are treated differently based on how we


use them. Here are some examples:

I saw the road sign AH26 when I travelled from Manila to Isabela.
Jonathan is number 26 in the Mathematics class.
I gained 26 points from my recent purchase.

All three sentences used the number 26 in different contexts. Can


you tell the difference?

The first sentence talks about a road sign AH26 (Pan-Philippine


Highway) which means Asian Highway 26 of the Asian Highway
Network. In this case, the number 26 was used to describe and
distinguish the highway from other Asian highways. It does not tell

Page 9 of 31
that AH26 is a better highway than other Asian Highways. This means
that the number 26 in the first sentence is of nominal type.

The second sentence tells us that the number 26 is used to identify


Jonathan’s standing in the Mathematics class relative to his
classmates. In this case, an order or rank is considered and one can
compare the number 26 from the other numbers. Thus, for example,
Liza is number 13 in this class, then we can say that Liza performs
better than Jonathan. This the number 26 is of the ordinal type.

The third sentence tells us that the number 26 is used to describe


the literal number of points gained in a recent purchase. Here we
observe the use of a unit of measure which is points. Thus, if one gains
30 points from a recent purchase, we can say that this value is larger
than 26 points by 4 points. Thus, the number 26 in the third sentence
is of cardinal type.

With the given examples, it must be clear to use that we need to


have the skill of identifying the type of number that is being used in as
sentence.

Learning Activity 5

Direction. Identify the type of number used in each of the following


sentences: nominal (N), ordinal (O), cardinal (C)

Examples:

Chanda is the 4th child in the family. O


Gino has student number 2020-3214 N

Start Here:

1. His situation is a catch-22.


2. I have 5 pesos left in my wallet.
3. Myla has an overload of 4 teaching units.
4. William is 2nd in line to the throne.
5. I could not tell him that my mobile number is
09021112222.
6. Cromwell is so near to being top 2 in the
Regional Statistics Quiz.
7. Sheila is on her 5th month of pregnancy.
8. Jacky sold 21 tubs of pansit Cabagan.
9. Lessons become harder once you reach 3rd year
in this program.
10. Sunday is the 1st day of the week.

Page 10 of 31
3. The words “and” and “or” mean differently in mathematics from its
English use.

Let us consider the mathematical sentence

Two plus two equals four.

as a sentence of English rather than of mathematics, and try to analyze


it grammatically. The sentence contains three nouns two, two, and four,
a verb equals, and a conjunction plus. Here we observe that the word
plus resembles the word and, the standard example of a conjunction.
However, we also observe that it does not behave in the same manner,
as is shown by the sentence,

Mario and Dana love Mathematics.

The verb in this sentence, love, is plural, while the verb in the
previous sentence, equals was singular. So the word plus seems to take
two objects and produce out of them a new, single object four, while and
conjoins Mario and Dana in a looser way, leaving them as distinct
people.

We found out that the conjunction and has two very different uses.
One is to link two nouns whereas the other is to join two whole
sentences together, as in

Mario loves Mathematics and Dana loves English.

If we want our language to be absolutely clear, then it is important


to be aware of this distinction. Thus, the sentence

Two and five are prime numbers.

may be paraphrased to

Two is a prime number and five is a prime number.

4. Mathematical objects may be represented in many ways such as sets


and functions.

Representations using sets and functions will be discussed in


detail in the succeeding topics.

Page 11 of 31
2.3.5 Four Basic Concepts

The four basic concepts to be discussed include sets, relations, functions,


and binary operations. It is important to be familiar with these basic concepts
and understand how they are used.

2.3.5.1 Sets

A set is a collection of objects, mostly mathematical objects such as


numbers, points in space or other sets. We denote a set using capital letters
of the English alphabet and the symbol ϵ for set membership. Thus, if 𝐴 is a
set and 3 is an element of this set, we may write it in symbol as 3 ϵ 𝐴 which is
read 3 𝑖𝑠 𝑎𝑛 𝑒𝑙𝑒𝑚𝑒𝑛𝑡 𝑜𝑓 𝐴 or 3 𝑖𝑠 𝑖𝑛 𝐴.

A set may also be empty, that is, it has no elements. We call this set the
empty set or the null set. The symbols used to denote the empty set are ∅ and
{ }.

There are three common ways to denote a specific set.

1. Roster method. Some sets have few elements which makes it possible
to provide a list of its members. One way of denoting a set is by the
roster method where we list its elements inside curly brackets:
{2, 3, 5, 7, 11, 13, 17, 19}, for example, is the set whose elements are the
eight numbers 2, 3, 5, 7, 11, 13, 17, and 19.

2. Use of dots. Most sets are too large and they are often infinite. A second
way to denote sets is to use three dots to imply a list that is too long to
write down: for example, the expressions {1, 2, 3, … , 100} and {2, 4, 6, 8, … }
represent the set of all positive integers up to 100 and the set of all
positive even numbers, respectively. The three dots are collectively
called ellipsis.

3. Rule method. A third way is to define a set using a property or a


description. An example that shows how this is done is the expression
{𝑥 ∶ 𝑥 𝑖𝑠 𝑝𝑟𝑖𝑚𝑒 𝑎𝑛𝑑 𝑥 < 20}. To read this expression, we first say, The set
of, because of the curly brackets. Next, we read the symbol that occurs
before the colon. The colon itself we read as such that. Finally, we read
what comes after the colon, which is the property that determines the
elements of the set. In this example, we say, The set of all x such that x
is prime and x is less than 20, which is equal to the set
{2, 3, 5, 7, 11, 13, 17, 19}.

Many sentences of mathematics can be written in set-theoretic terms.


For example, the sentence 5 < 10 could be written as 5 ϵ {𝑛 ∶

Page 12 of 31
𝑛 𝑖𝑠 𝑎 𝑛𝑎𝑡𝑢𝑟𝑎𝑙 𝑛𝑢𝑚𝑏𝑒𝑟 𝑎𝑛𝑑 𝑛 < 10}. Why is there a need for such if it is easier
to understand 5 < 10? In some instances, doing so would be convenient.

An example of this is when we translate geometry to algebra using


Cartesian coordinates. Geometric objects were defined as sets of points,
where points were expressed as pairs or triples of numbers. For example, the
set {(𝑥, 𝑦) ∶ 𝑥 2 + 𝑦 2 = 1} represents a circle of radius 1 about the origin (0, 0).
This was based on Pythagorean theorem which says that the square of the
distance from (0, 0) to (𝑥, 𝑦) is 𝑥 2 + 𝑦 2 so the sentence 𝑥 2 + 𝑦 2 = 1 can be re-
expressed geometrically as the distance from (0, 0) to (𝑥, 𝑦) is √1 = 1.

Sets allow one to reduce the number of parts of speech that one needs,
turning almost all of them into nouns. For example, with the help of the
membership symbol ϵ one can do without adjectives. The translation of 5 is a
prime number (where “prime” functions as an adjective) will simply be 5 ϵ 𝑃.

Given two sets 𝐴 and 𝐵, it is possible to “multiply” them to produce a new


set denoted as 𝐴 × 𝐵. This operation is called the Cartesian product. To
understand it, we must first understand the idea of an ordered pair.

DEFINITION 2.3 (ordered pair)

An ordered pair is a list (𝑥, 𝑦) of two things 𝑥 and 𝑦, enclosed in parentheses and
separated by a comma.

We take note that (2,4) is an ordered pair different from another ordered
pair (4,2). We write (2,4) ≠ (4,2). Right away you can see that ordered pairs
can be used to describe points on the plane, as was done in calculus, but they
are not limited to just that. Letters, for example, may be expressed as ordered
pairs like (𝑚, 𝑙). Now we are ready to define the Cartesian product.

DEFINITION 2.4 (Cartesian product)

The Cartesian product of two sets 𝐴 and 𝐵 is another set, denoted as 𝐴 × 𝐵 and
defined as 𝐴 × 𝐵 = {(𝑎, 𝑏) ∶ 𝑎 ∈ 𝐴, 𝑏 ∈ 𝐵} .

The set 𝐴 × 𝐵 is a set of ordered pairs of elements from 𝐴 and 𝐵. For


example, if 𝐴 = {𝑘, 𝑙, 𝑚} and 𝐵 = {1,2},then

𝐴 × 𝐵 = {(𝑘, 1), (𝑘, 2), (𝑙, 1), (𝑙, 2), (𝑚, 1), (𝑚, 2)}

Page 13 of 31
Learning Activity 6

Direction. Write the following sets using the rule method.

Example

{2,4,6,8,10} {2𝑛 ∶ 𝑛 ϵ ℤ , 1 ≤ 𝑛 ≤ 5}

Start Here:

1. The first 12 natural numbers.


2. {3,6,9,12,15, … }
3. {1,4,9,16,25,36, … }
4. {… , −15, −10, −5,0,5,10,15, … }
5. {… , −45, −30, −15,0,15,30,45, … }

Sets have other properties. It can happen that all elements of some set
𝐴 are also elements of another set 𝐵. For example, each element of 𝐴 = {1,3,4}
is also an element of 𝐵 = {0,1,2,3,4}. When 𝐴 and 𝐵 are related this way we say
that 𝐴 is a subset of 𝐵.

DEFINITION 2.5 (subset)

Suppose 𝐴 and 𝐵 are sets. If every element of 𝐴 is also an element of 𝐵, then we


say 𝐴 is a subset of 𝐵, and we denote this as 𝐴 ⊆ 𝐵.

We write 𝐴 ⊈ 𝐵 if 𝐴 is not a subset of 𝐵, that is, if it is not true that every element
of 𝐴 is also an element of 𝐵. Thus 𝐴 ⊈ 𝐵 means that there is at least one element
of 𝐴 that is not an element of 𝐵.

Just as numbers are combined with operations such as addition,


subtraction and multiplication, there are various operations that can be
applied to sets. The Cartesian product is one such operation; given sets 𝐴 and
𝐵, we can combine them with × to get a new set 𝐴 × 𝐵. Here are three
operations called union, intersection and difference.

DEFINITION 2.6 (set union, intersection, and difference)

Suppose 𝐴 and 𝐵 are sets.

 The union of 𝐴 and 𝐵 is the set 𝐴 ∪ 𝐵 = { 𝑥 ∶ 𝑥 ∈ 𝐴 or 𝑥 ∈ 𝐵 }.


 The intersection of 𝐴 and 𝐵 is the set 𝐴 ∩ 𝐵 = { 𝑥 ∶ 𝑥 ∈ 𝐴 and 𝑥 ∈ 𝐵 }.
 The difference of 𝐴 and 𝐵 is the set 𝐴 − 𝐵 = { 𝑥 ∶ 𝑥 ∈ 𝐴 and 𝑥 ∉ 𝐵} .

In words, the union 𝐴 ∪ 𝐵 is the set of all things that are in 𝐴 or in 𝐵 (or in both).
The intersection 𝐴 ∩ 𝐵 is the set of all things in both 𝐴 and 𝐵. The difference 𝐴 − 𝐵
is the set of all things that are in 𝐴 but not in 𝐵.

Page 14 of 31
2.3.5.2 Relations

Mathematical phrases such as “equals”, “is less than”, and “is an element
of” are some examples of relationships in mathematics. It is important, when
specifying a relationship, to be careful about which objects are to be related.
The term relation is used to describe a relationship between two mathematical
objects. Usually a relation comes with a set 𝐴 of objects that may or may not
be related to each other. For example, the relation < might be defined on the
set of all positive integers. In this case, we say 5 < 10, which satisfy the
defined relationship.

Sometimes relations are defined with reference to two sets 𝐴 and 𝐵. For
example, if the relation is ϵ, then 𝐴 might be the set of all positive integers and
𝐵 the set of all sets of positive integers as well. The formal definition of relation
follows.

DEFINITION 2.7 (relation)

A relation on a set 𝐴 is a subset 𝑅 ⊆ 𝐴 × 𝐴. We often abbreviate the statement


(𝑥, 𝑦) ∈ 𝑅 as 𝑥𝑅𝑦. The statement (𝑥, 𝑦) ∉ 𝑅 means 𝑥 is not related to 𝑦.

A relational expression 𝑥𝑅𝑦 is a sentence which is either true or false. For


example, 5 < 10 is true, and 10 < 5 is false. Thus an operation like “+” is not
a relation, because, for instance, 5 + 10 has a numeric value, not a True or
False value.

Since relational expressions have true or false values, we can combine


them and form the sentence “If 𝑥𝑅𝑦, then 𝑦𝑅𝑥”. This is a sentence whose truth
or falsity may depend on 𝑥 and 𝑦. With this, we take note that some relations
have properties that others do not have. For example, the relation ≤ on the
set of integers (ℤ) satisfies 𝑥 ≤ 𝑥 for every 𝑥 ϵ ℤ. But this is not the case for <
because 𝑥 < 𝑥 is never true.

Page 15 of 31
Learning Activity 7

Directions. Write out the defined relation 𝑅 on a set 𝐴 as a set of ordered


pairs.
Example

Let 𝐴 = {0,1,2,3,4,5} and 𝑅 expresses > 𝑅 = {(1,0), (2,0), (3,0), (4,0), (5,0),
on 𝐴. (2,1), (3,1), (4,1), (5,1), (3,2),
(4,2), (5,2), (4,3), (5,3), (5,4)}
Note: (1,0) is in 𝑅 because 1 > 0.

Start Here:

1. Consider the set 𝐴 in the example.


Write out 𝑅 which expresses = on 𝐴.

2. Still on 𝐴 in the example, write out 𝑅


which expresses ≤ on 𝐴.

2.3.5.3 Functions

One of the most basic activities of mathematics is to take a mathematical


object and transform it into another one, sometimes of the same kind and
sometimes not. “The square root of” transforms numbers into numbers, as do
“four plus”, “two times”, “the cosine of” and “the logarithm of”. A non-numeric
example is “the center of gravity of”, which transforms geometrical shapes into
points - meaning that if S stands for a shape, then “the center of gravity of S”
stands for a point. A function is a mathematical transformation of such a
kind. More generally, functions can have properties and in order to discuss
those properties one needs to think of functions as things.

Consider the function 𝑓 (𝑛) = |𝑛| + 2 that converts integers 𝑛 into natural
numbers |𝑛| + 2. Its graph is 𝑅 = {(𝑛, |𝑛| + 2) ∶ 𝑛 ϵ ℤ} ⊆ ℤ × ℕ.

Figure 1
The function 𝑓 (𝑛) = |𝑛| + 2

Page 16 of 31
A function may be viewed as:

1. sending elements from one set 𝐴 to another set 𝐵. In the case of 𝑓 in


Figure 1, 𝐴 = ℤ and 𝐵 = ℕ.
2. a relation from 𝐴 to 𝐵.
3. for every input value 𝑛, there is exactly one output value 𝑓(𝑛).

Some of you may have used the vertical line test: Any vertical line
intersects a function’s graph at most once. It means that for any input value
𝑥, the graph contains exactly one point of form (𝑥, 𝑓(𝑥)). The formal definition
that follows captures all these ideas.

DEFINITION 2.8 (function)

Suppose 𝐴 and 𝐵 are sets. A function 𝑓 from 𝐴 to 𝐵 (denoted as 𝑓 ∶ 𝐴 → 𝐵) is a


relation 𝑓 ⊆ 𝐴 × 𝐵 from 𝐴 to 𝐵, satisfying the property that for each 𝑎 ∈ 𝐴 the
relation 𝑓 contains exactly one ordered pair of form (𝑎, 𝑏). The statement (𝑎, 𝑏) ∈ 𝑓
is abbreviated 𝑓(𝑎) = 𝑏.

Consider the function 𝑓 graphed in Figure 1. According to the definition,


we regard 𝑓 as the set of points in its graph, that is, 𝑓 = {(𝑛, |𝑛| + 2) ∶ 𝑛 ∈ ℤ} ⊆
ℤ × ℕ. This is a relation from ℤ to ℕ, and indeed given any 𝑎 ϵ ℤ the set
𝑓 contains exactly one ordered pair (𝑎, |𝑎| + 2) whose first coordinate is 𝑎.
Since (3,5) ϵ 𝑓, we write 𝑓 (3) = 5; and since (−3,5) ϵ 𝑓 we write 𝑓 (−3) = 5, etc.
In general, (𝑎, 𝑏) ϵ 𝑓 means that 𝑓 sends the input value 𝑎 to the output value
𝑏, and we express this as 𝑓(𝑎) = 𝑏. This function can be expressed by a
formula: For each input value 𝑛, the output value is |𝑛| + 2, so we may write
𝑓 (𝑛) = |𝑛| + 2.

Learning Activity 8

Directions. Write out the defined function 𝑓 on the given sets as a set of
ordered pairs.
Example

There are four different functions 𝑓 = {(𝑎, 0), (𝑏, 0)}


𝑓: {𝑎, 𝑏} → {0,1}. List them all. 𝑓 = {(𝑎, 1), (𝑏, 1)}
𝑓 = {(𝑎, 0), (𝑏, 1)}
𝑓 = {(𝑎, 1), (𝑏, 0)}

There are eight different functions


𝑓: {𝑎, 𝑏, 𝑐 } → {0,1}. List them all.

Page 17 of 31
Some important concepts of functions include domain, codomain, and
range. Their definitions are given below.

DEFINITION 2.9 (domain and range)

For a function 𝑓 ∶ 𝐴 → 𝐵, the set 𝐴 is called the domain of 𝑓 . The set 𝐵 is called
the codomain of 𝑓 . The range of 𝑓 is the set {𝑓 (𝑎): 𝑎 ∈ 𝐴} = {𝑏 ∶ (𝑎, 𝑏) ∈ 𝑓} .

 Think of the domain as the set of possible “input values” for 𝑓.


 Think of the codomain as a sort of “target” for the outputs.
 Think of the range as the set of all possible “output values” for 𝑓.

Continuing the domain of 𝑓 is ℤ and its codomain is ℕ. Its range is


{𝑓 (𝑎): 𝑎 ∈ ℤ} = {|𝑎| + 2: 𝑎 ∈ ℤ} = {2, 3, 4, 5, … }. Notice that the range is inside
the codomain, but in this case, it is not equal to the codomain. In the example,
the domain and codomain are sets of numbers, but this need not be the case
in general.

Learning Activity 9

Directions. Write the domain and range of the given functions in set
notation.

Example

Suppose 𝐴 = { 0,1,2,3,4 }, 𝐵 = {2,3,4,5} Domain of 𝑓 = {0,1,2,3,4}


and 𝑓 = {(0,3), (1,3), (2,4), (3,2), (4,2)}. Range of 𝑓 = {2,3,4}
State the domain and range of 𝑓.

Start Here:

Suppose 𝐴 = {𝑎, 𝑏, 𝑐, 𝑑} , 𝐵 = {2,3,4,5,6},


and 𝑓 = {(𝑎, 2), (𝑏, 3), (𝑐, 4), (𝑑, 5)} . State
the domain and range of 𝑓.

2.3.5.4 Binary Operations


Consider the sentence “two plus two equals four”. It was clear to us that
the word “equals” indicates a relation and it sits between the noun phrases
“two plus two” and “four” which then forms a sentence. But what about
“plus”? This word also sits between two nouns. However, the result, “two plus
two” is not a sentence but a noun phrase. This pattern is characteristic of
binary operations. Some familiar examples of binary operations are “plus”,
“minus”, “times”, and “divided by”.
As with functions, it is customary, and convenient, to be careful about
the set to which a binary operation is applied. A binary operation on a set 𝐴

Page 18 of 31
is a function that takes pairs of elements of 𝐴 and produces elements of 𝐴
from them. It is a function with the set of all pairs (𝑥, 𝑦) of elements of 𝐴 as its
domain and with 𝐴 as its range. The symbol for the operation comes between
𝑥 and 𝑦 rather than before them: we write 𝑥 + 𝑦 rather than +(𝑥, 𝑦).
Now that we have discussed some conventions in mathematics, it is
essential to also acquire the skill of translating phrases and sentences to
mathematical notations.

2.3.6 Translating Expressions and Sentences

The use of symbols in mathematics is essential in stating complex


thoughts and ideas with simplicity. In this way, some of the mathematical
problems are easily analyzed and solved. This makes translation to
mathematical symbols and notations a required skill in understanding and
studying mathematics.
In translating phrases and sentences to mathematical symbols, we follow
the given steps below:

Step 1: Identify the variable/s and constant/s

Step 2: Determine the connectors.

Step 3: Perform the translation of the phrase or sentence into symbols.

An example is the sentence “Seven and a number is fifteen.”

Seven And a number is fifteen


7 + 𝑥 = 15

We must be familiar with the keywords to proceed with translation. The


table below shows some key words and phrases that are used to describe
common mathematical operations. To write mathematical expressions and
sentences, we assign a variable to represent the unknown number. Common
representation of the unknown number is the letter 𝑥. However, other letters
and characters may also be used.

Page 19 of 31
Operation or
Keyword/Phrase Example Translation
Connector
Plus A number plus seven 𝑥+7

More than Five more than a number 5+𝑥

Addition The sum of The sum of a number and six 𝑥+6

(+) The total of The total of ten and a number 10 + 𝑥

Increased by A number increased by two 𝑥+2

Added to Eleven added to a number 𝑥 + 11

Minus A number minus eight 𝑥−8

Less than Four less than a number 𝑥−4

Subtraction The difference of The difference of a number and ten 𝑥 − 10

(−) Less Nine less a number 9−𝑥

Decreased by A number decreased by eight 𝑥−8

Subtracted from Six subtracted from a number 𝑥−6

Times Five times a number 5𝑥

The product of The product of five and a number 5𝑥


Multiplication
Twice; double Twice a number; double a number 2𝑥
(×)
Multiplied by A number multiplied by negative two −2𝑥
3
Of Three-fifths of a number 𝑥
5
𝑥
The quotient of The quotient of a number and nine
9
Division
20
Divided by Twenty divided by a number
(÷,/) 𝑥
𝑥
The ratio of The ratio of a number and twelve
12
The square;
Powers The square of a number; a number squared 𝑥2
squared

(𝑥 𝑛 ) The cube of;


The cube of a number; a number cubed 𝑥3
cubed

Equals Ten less than a number equals one. 𝑥 − 10 = 1

Is Three times a number is negative five. 3𝑥 = −5


Equals
Is the same as Eighteen is the same as twice the number. 18 = 2𝑥
(=)
Yields Twelve added to a number yields eleven. 𝑥 + 12 = 11
Nine less than a number amounts to
Amounts to 𝑥 − 9 = 25
twenty-five.

Page 20 of 31
In translation problems, the words sum, total, difference, product and
quotient imply at least two parts – use parentheses when a sum or difference
is multiplied. For example, the phrase "the sum of three times a number and
five" translates to "3𝑥 + 5," while the phrase "three times the sum of a number
and five" translates to "3(𝑥 + 5)."
Other common issues in converting English phrases and sentences to
mathematical expressions and sentences are summarized in the following
table.

Issues Incorrect Process Correct Process

Using the wrong key word for the One half of twenty
problem 1 1 20
20 ÷ ∗ 20 or
2 2 2

Incorrect word order with subtraction Five less than two


times a number
2𝑥 – 5
5 – 2𝑥

Not validating by translating from Twice the sum of three


symbols back to words. and a number
2(3 + 𝑥)
2 × 3 + 𝑥

Use of punctuation Three times a number, 3𝑥 + 4


plus four.

3(𝑥 + 4)

Redundancy 2𝑥 = 12 Twice a number is


Twice the product of twelve.
two and x is twelve.

Page 21 of 31
Here are more examples of translations to mathematical symbols.

Example Translation
1
One-half is a rational number. 2
ϵℚ

 𝑥 ϵ {7𝑛 ∶ 𝑛 ϵ ℕ}
 𝑥 ϵ {7𝑛 ∶ 𝑛 ϵ ℤ}
The number 𝑥 is a multiple of seven.
 𝑥 = 7𝑛 for some 𝑛 ϵ ℤ
 𝑥 = 7𝑛, 𝑛 ϵ ℤ
 𝑥 ϵ 𝐴 and 𝑥 ϵ 𝐵
The element 𝑥 belongs to both sets 𝐴 and 𝐵.  𝑥𝜖𝐴∩𝐵

The values of the natural number 𝑛 range  {−3, −2, −1, 0, 1, 2, 3, 4, 5, 6, 7, 8}


from negative three to eight.  {𝑛 ϵ ℕ ∶ −3 ≤ 𝑛 ≤ 8}

The values of the real number 𝑛 range from  {𝑛 ϵ ℝ ∶ −3 ≤ 𝑛 ≤ 8}


negative three to eight.  [−3,8]
The square of a number is not more than
𝑦 2 ≤ 20
twenty.
The square of a number in non-negative. 𝑥2 ≥ 0
𝑥 + 8 = 2𝑦
where
Tina’s age in eight years is twice Gino’s age.
𝑥 is Tina’s present age and
𝑦 is Gino’s present age

Learning Activity 10

Directions. Translate the following sentences to mathematical symbols.

Start Here:

1. The sum of two numbers is seven.


2. Three less than a number is twelve.
3. Four more than a number is fifteen.
4. Five less than twice a number is ten.
5. Five-eighths of a number is twenty.
6. The values of the real number 𝑥 ranges from
negative four to seventeen.
7. Two is a real number.
8. The sum of the squares of two numbers.
9. The square of the sum of two numbers.
10. The cube of thrice a number added to five
yields thirty.

Page 22 of 31
2.3.7 Some Elementary Logic

Essential to the study of mathematics is logic. Aristotle is generally


regarded as the Father of Logic.
Logic started in the late 19th century with the development of axiomatic
frameworks for analysis, geometry and arithmetic. It allows us to determine
the validity of arguments in and out of mathematics. Through logic, the
preciseness and conciseness of mathematics can be illustrated.
2.3.7.1 Statement or Proposition
A statement (or proposition) in logic is a declarative sentence which is
either true or false, but not both. The truth value of the statements is the
truth and falsity of the statement.
Here are some examples to distinguish statements from other
sentences.
Sentence Remarks
1. Manila is the capital of the
The sentence is a true statement.
Philippines.
This is not a statement since it is not
2. What day is it?
declarative.
This sentence is not a statement since it
3. Help me, please.
cannot be categorized as true or false.
This sentence is not a statement since it is
4. He is handsome.
neither true nor false.
This is not a statement since it is not
5. Wait!
declarative.
6. All multiples of seven are odd This is a statement. However, this
numbers. statement is false.
This is not a statement since it is not
7. Sketch the graph of 𝑓(𝑥 ) = 2.
declarative.
8. The set of natural numbers is a
This is a true statement.
subset of itself.

Statements are of two types: simple and compound. A simple statement


is a statement that conveys a single idea. A compound statement is a
statement composed of two or more simple statements connected by some
words and phrases such as and, or, if … then, and if and only if creates a
compound statement.
For instance, the statement “I will attend the meeting or I will go to
school.” is a compound statement. It is composed of two simple statements,
“I will attend the meeting.” I will go to school.” The word or connects the two
simple statements. In logic, the word “or” is a logical connective.

Page 23 of 31
Learning Activity 11

Directions. Identify whether or not each of the given sentences is a


statement: statement (S), not a statement (NS)

Start Here:

1. Write the symbol for addition.


2. Two is an even number.
3. Where is the triangle in the set?
4. Five less than twice a number is ten.
5. This is how you solve 3𝑥 = 5.

2.3.7.2 Logical Connectives and Symbols


In logic, variables are used to represent statements. A formal
propositional statement is written using propositional logic notations p, q, and
r which are used to represent simple statements and the symbols , , ~, →
and ↔ to represent logical connectives.
The following table summarizes the different symbols logical connectives and
how they are applied to statements.

Type of Symbolic
Statement Connective Truth value
statement form
True if 𝑝 and 𝑞 are
Conjunction 𝑝 and 𝑞 And 𝑝𝑞
both true
True if either 𝑝 or 𝑞 is
Disjunction 𝑝 or 𝑞 Or 𝑝𝑞
true
True except when 𝑝 is
Conditional If 𝑝, then 𝑞 if … then 𝑝 → 𝑞
true and 𝑞 is false.
True if 𝑝 and 𝑞 are
Biconditional 𝑝 if and only if 𝑞 if and only if 𝑝 ↔ 𝑞 both true of if 𝑝 and 𝑞
are both false

2.3.7.3 Negation

The negation of the statement is its opposite. For example, the negation
of the statement “Five is a rational number” is the statement “Five is not a
rational number.” The tilde symbol (~) is used to denote the negation of a
statement. If the statement 𝑝 is true, its negation ~𝑝 is false, and if the
statement 𝑝 is false, its negation ~𝑝 is true. Meaning, the truth value of the
negation of a statement is always the reverse of the truth value of the original
statements.

Page 24 of 31
A popular theorem facilitates negation of compound statements of the
forms 𝑝  𝑞 and 𝑝  𝑞. It is called De Morgan’s Laws which are stated as
follows:

1. ~(𝑝  𝑞) is equivalent to ~𝑝  ~𝑞
2. ~(𝑝  𝑞) is equivalent to ~𝑝  ~𝑞

To get the negation of the statement “A number 𝑥 is greater than or equal


to a number 𝑦.”, we let

𝑝 ∶ A number 𝑥 is greater than a number 𝑦.


𝑞: A number 𝑥 is equals to a number 𝑦.

The statement in symbols is given by 𝑝  𝑞 with negation ~(𝑝  𝑞). By De


Morgan’s law, ~(𝑝  𝑞) is equivalent to ~𝑝  ~𝑞. We have

~𝑝: A number 𝑥 is not greater than a number 𝑦.


~𝑞: A number 𝑥 is not equal to a number 𝑦.
~𝑝  ~𝑞: A number 𝑥 is not greater than a number 𝑦 and
not equal to 𝑦.
~𝑝  ~𝑞: A number 𝑥 is less than a number 𝑦.

The statement ~𝑝: A number 𝑥 is not greater than a number 𝑦 means that
either 𝑥 is less than 𝑦 or 𝑥 equals. However, ~𝑞: A number 𝑥 is not equal to a
number 𝑦 tells us that 𝑥 cannot be equal to 𝑦. Thus, the negation is “ a
number 𝑥 is less than 𝑦.

2.3.7.4 Truth value and truth tables

The truth value of a simple statement is either true (T) or false (F) but not
both. For compound statements, the truth value depends on the truth values
of its simple statements and connectives.

The truth table is a table that shows the truth value of a compound
statement for all possible truth values of its simple statements. The following
table gives the truth values of compound statements with two simple
statements 𝑝 and 𝑞.

𝑝 𝑞 p𝑞 𝑝𝑞 𝑝 → 𝑞 𝑝 ↔ 𝑞
T T T T T T
T F F T F F
F T F T T F
F F F F T T

Page 25 of 31
2.3.7.5 Writing Compound Statements in symbols

To write a compound statement in symbols, we need to identify its simple


statements and represent them using the symbols 𝑝, 𝑞, and 𝑟.

Consider the compound statement “Two and three are rational zeroes of
the function 𝑓.” Here, the word “and” suggests that the compound statement
may be paraphrased as “Two is a rational zero of 𝑓 and three is a rational zero
of 𝑓.” Thus, we may represent the simple statements as

𝑝: Two is a rational zero of 𝑓.


𝑞: Three is a rational zero of 𝑓.

For the connector “and”, we use the symbol . Thus, the statement in
symbol is given by 𝑝  𝑞.

The compound statement “If an integer is odd, then it is not divisible by


2.” Has simple statements

𝑝 : An integer is odd.
𝑞 : An integer is not divisible by 2.

Since the compound statement is a conditional statement, we use the


logical connective →. Hence, the statement in symbol is given by 𝑝 → 𝑞.

Learning Activity 12

Directions. Identify the simple statements in each of the following


compound statements. Then write the compound statement in symbols.

Example:
𝑛 𝑛
If 2 is an integer, then 𝑛 is an 𝑝: 2 is an integer. 𝑝→𝑞
even integer. 𝑞: 𝑛 is an even integer
Start Here:

1. The numbers 2 and −3 are 𝑝:


rational zeros of the function 𝑓. 𝑞:

2. If 𝑥 is even, then it is a multiple 𝑝:


of 2. 𝑞:

3. Either 𝑓 is an odd or 𝑝:
even function. 𝑞:

Page 26 of 31
2.3.8 Quantifiers

A predicate (or open statement) is a statement whose truth depends on


the value of one or more variables. Predicates become propositions once every
variable is bound by assigning a universal set.

An example is the sentence “𝑥 is an even number”. It is a predicate whose


truth depends on the value of 𝑥. Using the notation 𝑃(𝑥) to represent the “𝑥
is an even number.”, we know that 𝑃(2) is true, and 𝑃(3) is false. Here, 𝑃(𝑥)
is called a propositional function which becomes a statement only when
variable 𝑥 is given particular value. Propositional functions are usually
denoted as 𝑃(𝑥 ), 𝑄(𝑥 ), and 𝑅(𝑥).

In a statement, the word some and the phrases there exists and at least
one are called existential quantifiers. Existential quantifiers are used as
prefixes to assert the existence of something. In a statement, the words none,
no, all, and every are called universal quantifiers. The universal quantifiers
none and no deny the existence of something, whereas the universal
quantifiers all and every are used to assert that every element of a given set
satisfies some condition. The following table shows the symbols for universal
and existential quantifiers and their translations.

Quantifier Symbol Translation


Existential ∃ There exists
There is some
For some
For which
For at least one
Such that
Satisfying
Universal ∀ For all
For each
For every
For any
Given any

The statement “there exists an 𝑥 such that 𝑃(𝑥),” is symbolized by


∃𝑥, 𝑃(𝑥). The symbol ∃ is used to denote the existential quantifier. The
statement “∃𝑥, 𝑃(𝑥)”is true if there is at least one value of 𝑥 for which 𝑃(𝑥) is
true.

The statement “for all 𝑥, 𝑃(𝑥),” is symbolized by ∀𝑥, 𝑃(𝑥). The symbol 
is used to denote the universal quantifier. The statement “∀𝑥, 𝑃(𝑥)”is true if
only if 𝑃(𝑥) is true for every value of 𝑥.

Page 27 of 31
The truth values for quantified statements are given in the following
table.

Statement Is True when Is False when


There is at least one 𝑥 for
∃𝑥, 𝑃(𝑥) 𝑃(𝑥) is true for every 𝑥.
which 𝑃(𝑥) is false.
There is at least one 𝑥 for
∀𝑥, 𝑃(𝑥) 𝑃(𝑥) is false for every 𝑥.
which 𝑃(𝑥) is true.

The following English statements are paired with their translations into
symbolic form. Their truth values are also included. It must be clear that 𝐸(𝑛)
represents the set of even integers and 𝑂(𝑛) the set of odd integers.
Statement Translation Truth Value
Every integer that is not odd is  ∀𝑛 ∈ ℤ, ∼ (n is odd ) → True
even. (n is even)
 ∀𝑛 ∈ ℤ, ∼ 𝑂(𝑛) → 𝐸(𝑛)
There is an integer that is not  ∃𝑛 ∈ ℤ, ∼ 𝐸(𝑛) True
even.
For every real number 𝑥, there  ∀𝑥 ∈ ℝ, ∃𝑦 ∈ ℝ, 𝑦 3 = 𝑥 True
is a real number 𝑦 for which 𝑦
3 = 𝑥.
Given any two rational  ∀𝑎, 𝑏 ∈ ℚ, 𝑎𝑏 ∈ ℚ True
numbers 𝑎 and 𝑏, it follows
that 𝑎𝑏 is rational.
Every integer is even.  ∀𝑛 ∈ ℤ, 𝐸(𝑛) False
There is an integer 𝑛 for which  ∃𝑛 ∈ ℤ, 𝑛2 = 2 False
𝑛2 = 2.
For every real number 𝑥, there  ∀𝑥 ∈ ℝ, ∃𝑦 ∈ ℝ, 𝑦 2 = 𝑥 False
is a real number 𝑦 for which
𝑦 2 = 𝑥.
Given any two rational  ∀𝑎, 𝑏 ∈ ℚ, 𝑝 𝑎𝑏 ∈ ℚ False
numbers a and b, it follows
that √𝑎𝑏 is rational.

Given a set 𝑆, a quantified statement of form ∀𝑥 ∈ 𝑆, 𝑃(𝑥) is understood to


be true if 𝑃(𝑥) is true for every 𝑥 ∈ 𝑆. If there is at least one 𝑥 ∈ 𝑆 for which
𝑃(𝑥) is false, then ∀𝑥 ∈ 𝑆, 𝑃(𝑥) is a false statement. Similarly, ∃𝑥 ∈ 𝑆, 𝑃(𝑥) is
true provided that 𝑃(𝑥) is true for at least one element 𝑥 ∈ 𝑆; otherwise it is
false.
We recall that the negation of a false statement is a true statement and
the negation of a true statement is a false statement. We will use this fact to
form the negation of quantified statements.

If we consider the statement, “All dogs are mean.”, we may think that the
negation is “No dogs are mean.”, but this is also a false statement. Thus the
statement “No dogs are mean.” is not the negation of “All dogs are mean.” The
negation of “All dogs are mean,” which is a false statement, is in fact “Some
dogs are not mean,” which is a true statement. The statement “Some dogs are
not mean” can also be stated as “At least one dog is not mean” or “There exists

Page 28 of 31
a dog that is not mean.” What is the negation of the false statement, “No
doctors write in a legible manner”?

When a statement contains two quantifiers, we must consider their order,


for reversing the order can change the meaning. Quantified statements are
often misused in casual conversation. Some would say “All students do not
pay full tuition.” when they mean “Not all students pay full tuition.” This
mistake must never be observed in a mathematical context. Do not say “All
integers are not even.” because that means there are no even integers.
Instead, say “Not all integers are even.”

Learning Activity 13

Directions. Write the following as English sentences. Say whether they are
true(T) or false(F)

Example:
For all real number 𝑥, the negative of 𝑥 False
∀𝑥 𝜖 ℝ, −𝑥 < 0. is less than zero.

Start Here:

1. ∀𝑥 𝜖 ℝ, 𝑥 2 ≥ 0

2. ∀𝑥 𝜖 ℝ, ∃𝑛 𝜖 ℕ, 𝑥 𝑛 ≥ 0

3. ∃𝑎 𝜖 ℝ, ∀𝑥 𝜖 ℝ, 𝑎𝑥 = 𝑥

2.4 Recommended learning materials and resources


 Mathematical Notation – Language of Mathematics by Sarada Herke
https://www.youtube.com/watch?v=Y-c_CgxxPF0
 How to Read Math from Domain of Science
https://www.youtube.com/watch?v=Kp2bYWRQylk
 Mathematical symbols
https://www.rapidtables.com/math/symbols/Basic_Math_Symbols.h
tml#basic
2.5 Flexible Teaching Learning Modality (FTLM)
Remote (asynchronous)

 module, exercises, problems sets, powerpoint lessons

Page 29 of 31
2.6 Assessment Task
A. Write your answer on the blank after each item.

1. Describe the set {6,8,10,12,14,16,18} using


rule method
2. What is the Cartesian product of the sets
{0,1,2} and {𝑎, 𝑏}?
3. Consider the set 𝐵 = {1,2,3,4,5}. The pair
(𝑥, 𝑦) ϵ 𝑅 if 𝑥 and 𝑦 are both odd or both
even. Write out 𝑅 as a set of ordered pairs.
4. Let 𝑓: {0,1,2} → {0,2,4} defined by 𝑓 (𝑎) =
2𝑎, 𝑎 ϵ {0,1,2}. Express 𝑓 as a set of ordered
pairs.
5. A visor costs 90 pesos more than a paper
fan. If we let ℎ be the cost of the fan, what
will be the cost of the visor in mathematical
symbols?
6. Jenny is older than Miko by 4 years. The
sum of their age is 45. If we let 𝑚 be Miko’s
age, what is the representation of the sum of
their ages in mathematical symbols?
7. What is the negation of the compound
statement: “The real number 𝑦 is less than
or equal to the real number 𝑥.”
8. Write the simple statements in the
compound statement: “If the parallelogram
has perpendicular diagonals, then it is a
rhombus.”
9. Write in symbols: “For every positive integer
𝑥, there exists a real number 𝑦 such that the
square of 𝑦 is 𝑥.”
10. Write in symbols: “There exist natural
numbers 𝑥 and 𝑦 such that the difference of
𝑥 and 𝑦 is the same as the difference of 𝑦
and 𝑥.”

B. Essay:
1. What is the nature of mathematics as a language?
2. What is the most useful about the language of mathematics?
3. Give your reaction to the following statements:
a. Mathematics is not a language, but a useless set of formal rules and
alien symbols.
b. Mathematics confuses the communication of concepts and ideas.
c. Mathematics is full of unnecessary symbols, rules, and conventions.

Page 30 of 31
Your answers in Essay will be graded according to the given
standards/basis for grading:
Score Criteria
Unable to elicit the ideas and concepts from the learning activity, material,
0
or video
Able to elicit the ideas and concepts from the learning activity, material, or
1
video but shows erroneous understanding
Able to elicit the ideas and concepts from the learning activity, material, or
2
video and shows correct understanding
Able to elicit the correct ideas from the learning activity, material, or video
3 and also shows evidence of internalization and consistently contributes
additional thought to the core idea

2.7 References
Books and Lecture Notes
Aufmann, et. al. 2013. Mathematical Excursions. Third Edition. Cengage
Learning, USA
Fischer, Carol Burns. The Language of Mathematics. One Mathematical Cat
Please.
Hammack, Richard. 2013 Book of Proof. Department of Mathematics and
Applied Mathematics. Virginia Commonwealth University
Jamison, R.E. 2000 Learning the Language of Mathematics. Language and
learning across the disciplines
Sirug, Winston. (2018) Mathematics in the Modern World. Mindshapers Co.,
Inc. Publishing Company
Internet source:
https://www.dpmms.cam.ac.uk/~wtg10/grammar.pdf
date: 08-17-2020

Page 31 of 31
Module 3
Problem Solving
and
Reasoning
MODULE 3
The Nature of Mathematics: Problem Solving and Reasoning

3.1 Introduction

Central to the study of mathematics is its ability to solve problems even


outside the field. Different professions deal with different problems. Most
professionals are required to have good problem solving skills. In the previous
module, you built a clear understanding of mathematics as a language and
improved your skills in translating mathematical sentences to symbols. This
skill is so important for you to become a better problem solver.

In this module, our main focus is on how you solve a problem in a


systematic manner.
3.2 Learning Outcomes

After finishing this module, you are expected to

1. discuss ways to solve a problem;


2. solve problems using inductive and deductive reasoning; and
3. solve problems using Polya’s strategy.

3.3 What You Need to Know

A mathematical problem is a situation that you may confront for which


the method and solution is not immediately known to you. There are different
ways to come up with the solution of a problem. Good comprehension and
reasoning are necessary.

3.3.1 Inductive and Deductive Reasoning

We start by describing two processes of reaching a conclusion: inductive


and deductive reasoning.

3.3.1.1 Inductive Reasoning

The type of reasoning that comes up to a conclusion by examining


specific examples is called inductive reasoning. A conclusion that is made by
applying inductive reasoning is called a conjecture. Conjectures may or may
not be correct.

An example of this type of reasoning is applied when we want to find the


next number in a list of numbers which follows a pattern.

Example 1. Use inductive reasoning to predict the next number in the lists.

1, 3, 6, 8, 11, ?
Solution.

The first two numbers differ by 2, the second and the third by 3, the third
and fourth by 2 again. It appears that when two numbers differ by 2, the next
difference would be 3, followed again by 2, then by 3. Since the difference
between 8 and 11 is 3, we predict the number next to 11 to be a number 2
more than 11, which is 13.

We used inductive reasoning when we predicted the number next to 11.

Example 2. Use inductive reasoning to make a conjecture.

Perform the following steps:

Pick a number. Multiply it by 4. Add 6 to the product. Divide the sum by


2. Subtract 3.

Use this procedure in different numbers and conjecture on the resulting


number.

Solution.
We may construct a table to examine the results.
Original number Result
5 10
6 12
7 14
10 20
100 200

If you pick the number 5, the procedure will give a result of 10. For 6, it
will give 12; for 7, 14; for 10, 20; and for 100, 200. We observe that the
resulting numbers are twice the original number. Thus, we conjecture that
the process will produce a number which is twice the original.

We used inductive reasoning in making a conjecture in the previous


example. We must take note that though it can be helpful to use inductive
reasoning to arrive at a conclusion, extra care must be observed when
performing procedure. Conclusions based on inductive reasoning may not be
correct. The following example illustrates this instance.

Page 2 of 20
Example 3. Consider the polynomial 𝑛2 + 19𝑛 − 19 where 𝑛 is a counting
number.

If we conjecture on the results for 𝑛 ≥ 2, we first check on several values


of 𝑛.

𝑛 𝑛2 + 19𝑛 − 19
2 23
3 47
4 73
5 101
6 131
7 163

We notice that the results are prime numbers. Thus, we conjecture that
for 𝑛 ≥ 2, then 𝑛2 + 19𝑛 − 19 is a prime number. To test our conjecture, we
need to check results for other values. We have

𝑛 𝑛2 + 19𝑛 − 19
8 23
9 47
10 73
11 101
12 131

We see that the results are consistent and that our conjecture seems
correct. If we continue checking on the next results, we get 397, 443, 491,
541, 593, and 647. All these numbers are prime. However, if 𝑛 = 19, we get
703 which is the product of 19 and 37. This makes us conclude that our
conjecture is incorrect.

Example 4. Consider a circle and dots on the circle. What we do is connect


each dot to all other dots so that we draw all possible line segments. After
this, we count the number of regions formed inside the circle. The first five
results are given in Figure 3.1.

Figure 3.1
Maximum number of regions formed by connecting dots on a circle

Page 3 of 20
Results may be summarized in the following table.

Number of dots 1 2 3 4 5 6
Maximum number of regions 1 2 4 8 16 ?

Using the information in the table, we see that as the number of dots
increase, the number of regions inside the circle is doubled. Thus, we expect
that for 6 dots, there will be 16 ∙ 2 = 32 regions. However, if we perform the
procedure, we find out the only 31 regions are formed as shown in Figure 3.2.

Figure 3.2
Maximum number of regions formed by connecting 6 dots on a circle

In itself, inductive reasoning is not a valid method of proof. Observing


that a pattern exists in a number of situations does not mean that the pattern
is true for all situations. However, the inductive way is still valuable because
it allows us to form ideas on what we observed which leads us to come up
with a hypothesis.

Learning Activity 1

Direction. Use inductive reasoning to figure out the next number in


the list.

Start Here:

1. 4, 8, 12, 16, 20, 24, ____


2. 3, 5, 9, 15, 23, 33, ___
3. 80, 70, 61, 53, 46, 40, ____
3 5 7 9 11 13
4. , , , , , ,____
5 7 9 11 13 15

5. 2, 7, −3, 2, −8, −3, −13, −8, −18, ____

Page 4 of 20
Observed patterns and formed ideas are better understood when one
knows how to communicate these ideas into meaningful statements. The
ability to analyze and convey the ideas formed is an essential part of the
problem solving process. In the succeeding discussion, we recall concepts on
truth of statements and familiarize with how we can justify the truth or falsity
of a statement.

A statement is true if it is true for all cases. If you can find one case
where the statement is not true, then it is considered a false statement. The
instance for which the statement becomes false is called a counter-example.
One can verify that a statement is false by using a counter-example. This is
illustrated in the next example.

Example 5. If 𝑛 is an integer, then the absolute value of 𝑛 is greater than 0.

Solution.

Since 0 is an integer, we let 𝑛 = 0. Now, |𝑛| = |0| = 0 ≯ 0. We have found


a counter-example. Thus, the statement “If 𝑛 is an integer, then the absolute
value of 𝑛 is greater than 0.” is a false statement.

Learning Activity 2

Direction. Find a number that provides a counter-example to show that the


given statement is false.

Example:

1
∀ real numbers 𝑥, 𝑥 > .
𝑥
Answer:
1 1
Consider the real number 1. Then 1 ≯ since 1 = . We have
1 1
found a counter-example. Therefore, the given statement
1
“∀ real numbers 𝑥, 𝑥 > 𝑥.” is false.

Start Here:

1. ∀ real numbers 𝑥, 𝑥 3 ≥ 𝑥.

2. ∀ real numbers 𝑥, |𝑥 + 3| = |𝑥 | + 3.

Page 5 of 20
3.3.1.2 Deductive Reasoning

Deductive reasoning, unlike inductive reasoning, is a valid form used in


proving observations and arriving at conclusions. It is a process by which
one makes conclusions based on previously accepted general assumptions,
procedures, and principles.

To illustrate how deductive reasoning works, we will use the same


procedure we had in Example 2 and this time we use a deductive process.

Example 6. Use deductive reasoning to make a conclusion.

Procedure: Pick a number. Multiply the number by 4. Add 6 to the


product. Divide the sum by 2 then subtract 3.

Solution.

Procedure Symbolic representation


Pick a number 𝑛
Multiply by 4 4𝑛

Add 6 to the product 4𝑛 + 6


4𝑛 + 6
Divide the sum by 2 = 2𝑛 + 3
2
Subtract 3 2𝑛 + 3 − 3 = 2𝑛

If we let 𝑛 be the number, the procedure will give a result of 2𝑛. Thus,
we conclude that the procedure produces a number which is twice the original
number.

Some problems make use of the deductive process to arrive at a solution.


Logic puzzles, for example, are best solved by deductive reasoning using a
chart to display the given information.

Example 7. Solve this puzzle using deductive reasoning.

John, Jenny, Sheila, and Jason were recently elected as new class
officers (president, vice president, secretary, treasurer) in a National High
School. From the following clues, determine which position each holds.

1. Jason is younger than the president but older than the treasurer.

2. John and the secretary are both the same age, and they are the
youngest members of the group.

3. Sheila and the secretary are next-door neighbors.

Page 6 of 20
Solution.

From clue 1, Jason is neither the president nor the treasurer. We put a
mark “X” on positions Jason do not hold.

President Vice Secretary Treasurer


President
Jason X X
Jenny
Sheila
John

From clue 2, John is not the secretary. And since he is one of the
youngest in the group, he cannot be the president. From here, we also
conclude that Jason is not the secretary since he is older than the treasurer.
Thus, Jason must be the vice president and John must be the treasurer.

President Vice Secretary Treasurer


President
Jason X  X X
Jenny X X
Sheila X X
John X X X 

From clue 3, Sheila is not the secretary which leads us to conclude


that Jenny is the secretary. Since there are no other options, we conclude
that Sheila is the president.

President Vice Secretary Treasurer


President
Jason X  X X
Jenny X X  X
Sheila  X X X
John X X X 

Therefore, Sheila is the president, Jason is vice president, Jenny is the


secretary, and John is the treasurer.

Page 7 of 20
Learning Activity 3

Direction. Solve the following problems using deductive reasoning.


Let 𝑛 be the original number.

Example

Pick a number. Add 4 to the 1. 𝑛 + 4


number and multiply the sum 2. 3(𝑛 + 4) = 3𝑛 + 12
by 3. Subtract 7 from the
3. 3𝑛 + 12 − 7 = 3𝑛 + 5
product. Then subtract triple
of the original number from 4. 3𝑛 + 5 − 3𝑛 = 5
this difference. The procedure will always result to
the number 5.

Start Here:

1. Pick a number. Multiply the


number by 6 and add 8. Divide
the sum by 2, subtract twice
the original number, and
subtract 4.

3.3.2 Problem Solving with Patterns

3.3.2.1 Terms of a Sequence

An ordered list of numbers such as

5, 14, 27, 44, 65, …

is called a sequence. The numbers in a sequence that are separated by


commas are the terms of the sequence. In the given sequence, 5 is the first
term, 14 is the second, and so on. The three dots indicate that the sequence
continues beyond 65, which is the last term written. The use of subscript
notation is a convention to designate the nth term of a sequence. That is,

𝑎1 represents the first term of a sequence.

𝑎2 represents the second term of a sequence.

𝑎3 represents the third term of a sequence.


𝑎𝑛 represents the nth term of a sequence.

When we examine a sequence, the following questions are often asked:

 What is the next term?


 What formula or rule can be used to generate the terms?

Before we proceed to the types of sequences, we will study the basic


sequences and find the next term of a sequence using a difference table.

Page 8 of 20
A difference table is often used to show differences between successive
terms of the sequence. The following table is the difference table for the
sequence

2, 5, 8, 11, 14, …
Sequence 2 5 8 11 14
First difference 3 3 3 3

Looking at the sequence, we see that the difference between 5 and 2 is 3;


8 and 5 is also 3; and so on.

In this case, the first differences are all the same. Thus, if we use the
difference table above to predict the next number in the sequence, we shall
be working upward by adding 3 to 14. Thus, 14 + 3 = 17 is the next term of
the sequence. The following table shows how this is done.
Sequence 2 5 8 11 14 17
First difference 3 3 3 3 3

Consider the sequence below with its difference table.

5, 14, 27, 44, 65, …


Sequence 5 14 27 44 65
First difference 9 13 17 21
Second difference 4 4 4

From the table, we see that the first differences are not the same since
14 − 5 = 9, 27 − 14 = 13, 44 − 27 = 17, and 65 − 44 = 21. Thus, we check on the
second differences and observe if we see a pattern. In this case, the second
differences are all the same. We have 13 − 9 = 4, 17 − 13 = 4, and 21 − 17 = 4.
The common second difference is 4.

Since we have found a common second difference, we shall be working


upward to get 4 + 21 = 25 and then 25 + 65 = 90. The following table provides
the illustration on how we get the next term of the sequence. Thus, in this
example, the next term is 90.

Sequence 5 14 27 44 65 90
First difference 9 13 17 21 25
Second difference 4 4 4 4

Page 9 of 20
Example 8. Use a difference table to predict the next term in the sequence.

2, 7, 24, 59, 118, 207, …

Solution.
Sequence 2 7 24 59 118 207 332
First differences 5 17 35 59 89 125
Second differences 12 18 24 30 36
Third differences 6 6 6 6

In this case, we found common third difference of 6. Working upward,


we have 6 + 30 + 89 + 207 = 332. This is the next in the given sequence.

Learning Activity 4

Direction. Figure out the next number in the sequence using a


difference table as your guide.

Start Here:

1. 1, 7, 17, 31, 49, 71, …


2. 10, 10, 12, 16, 22, 30, …
3. −1, 4, 21, 56, 115,204, …
4. 9, 4, 3,12, 37, 84, …
5. 17, 15, 25, 53, 105, 187, …

3.3.2.2 nth-Term Formula for a Sequence

Consider the formula 𝑎𝑛 = 3𝑛2 + 𝑛. This formula defines a sequence and


provides a method for finding any term of the sequence. In particular, if we
want to find the first, second, and third terms, we only need to substitute the
values 1,2, and 3 to 𝑛. So,

𝑎1 = 3(1)2 + 1 = 4,

𝑎2 = 3(2)2 + 2 = 14,

𝑎3 = 3(3)2 + 3 = 30,

which means that the first term is 4; second term is 14; and third term is
30.

The next example illustrates how the 𝑛𝑡ℎ-term formula can be


determined.

Page 10 of 20
Example 9. Assume the pattern shown by the square tiles in the following
figure continues.

a. What is the 𝑛𝑡ℎ-term formula for the number of tiles in the 𝑛𝑡ℎ figure
of the sequence?
b. How many tiles are in the eighth figure of the sequence?
c. Which figure will consist exactly 320 tiles?

Solution.

a. If we count the tiles in each term, we get the sequence 2, 5, 8, 11, … which
has a common difference of 3. Thus, we may consider 3𝑛 such that 𝑛 =
1, 2, …. However, using this would give 3 as the first term.

From 3, we subtract 1 to get 2. Hence, we may consider the formula


3𝑛 − 1, where 𝑛 = 1,2, …. Evaluating this formula for the first 4 terms
will give us 2, 5, 8, and 11, which are the terms of the sequence based on
the figure.

Thus, the 𝑛𝑡ℎ-term formula for this sequence is 𝑎𝑛 = 3𝑛 − 1.

b. Using the formula we derived from (a), we can get the number of tiles
in the eighth term of the sequence. In this case, 𝑛 = 8 and 𝑎8 = 3(8) −
1 = 24 − 1 = 23 tiles.

c. To determine which figure in the sequence will have 320 tiles, we will
use the formula we derived in (a) and solve for 𝑛. Here, we expect the
𝑛𝑡ℎ-term to have 320 tiles. Thus, we solve 3𝑛 − 1 = 320.

3𝑛 − 1 = 320
3𝑛 = 321
𝑛 = 107

Therefore, the 107th figure has 320 tiles.

Do you have your own way of solving this problem? It will help if you
write your own solution and compare the results.

Page 11 of 20
3.3.2.3 Types of Sequences
There are different types of sequences in mathematics. The succeeding
discussion will illustrate some of these sequences.

Arithmetic Sequence

An arithmetic sequence is made by adding the same value each time.


This value is referred to as the ‘’common difference’’. A sequence with this
property is also called arithmetic progression.

The general form is given by

𝑎, 𝑎 + 𝑑, 𝑎 + 2𝑑, 𝑎 + 3𝑑, 𝑎 + 4𝑑, …

where 𝑎 is the first term and 𝑑 is the common difference.

Here are some examples of arithmetic progressions with their common


difference.

Example Common difference


1, 5, 9, 13, 17, 21, 25, 29, 33, . .. 4
4, 7, 10, 13, 16, 19, 22, 25, . .. 3
29, 27, 25, 23, 21, 19, . .. −2

The sequences used in the previous examples are arithmetic


progressions.

Geometric Sequence

We shall now move on to the other type of sequence we want to explore.


Consider the sequence

2, 4, 8, 16, 32, 64, …

Here, each term in the sequence is 2 times the previous term. A sequence
such as this is called a geometric sequence.

There is a general notation for geometric sequences. The first term we


shall denote by 𝑎. But here, there is no common difference. Instead there is
a common ratio, as the ratio of successive terms is always constant. So we
shall let 𝑟 be this common ratio.

With this notation, the general geometric sequence can be expressed as

𝑎, 𝑎𝑟, 𝑎𝑟 2 , 𝑎𝑟 3 , …

The 𝑛𝑡ℎ-term can be calculated using 𝑎𝑟 𝑛−1 , where the power (𝑛 − 1) is


always one less than the position 𝑛 of the term in the sequence.

Page 12 of 20
Thus, the sequence

2, 4, 8, 16, 32, 64, …

can be expressed as

2, 2(2)1 , 2(2)2 , 2(2)3 , 2(2)4 , 2(2)5 , …

Triangular Number Sequence

1, 3, 6, 10, 15, 21, 28, 36, 45, ...

The Triangular Number Sequence is generated from a pattern of dots


which form a triangle:

By adding another row of dots and counting all the dots we can find the
next number of the sequence. Thus, the 5th term in this sequence is:

Learning Activity 5

Direction. Find the 𝑛𝑡ℎ-term formula for the sequence with the given
representation below.

How many squares will there be in the fifteenth figure?

Page 13 of 20
3.3.3 Polya’s Problem Solving Strategy

A great discovery solves a great problem but there is a grain of discovery in the solution of any problem. Your problem may
be modest; but if it challenges your curiosity and brings into play your inventive faculties, and if you solve it by your own
means, you may experience the tension and enjoy the triumph of discovery.

George Polya

For a mathematician, the answer to a mathematical problem is


important. Equally important is the study of methods and rules for discovery
and invention. This is called heuristics.

One highly acceptable method to problem solving is designed by George


Polya (1887-1985). The basic problem-solving strategy that Polya advocated
consisted of four steps: understand the problem; devise a plan; carry out the
plan; and review the solution.

We will discuss each step by giving some guide on how we can complete
the problem solving process devised by George Polya.
3.3.3.1 Understand the Problem

We must have a clear understanding of the problem. In this part, we may


list the given information. Essential to this part is the ability to identify what
is asked and have a clear understanding of the properties of the final answer.

3.3.3.2 Devise a Plan

Different problems can be solved using a variety of solutions. Some of


these procedures include making a list of the known information, making a
list of information that is needed, sketching a diagram, making an organized
list that shows all possibilities, making a table or a chart, working backwards,
trying to solve a similar but simpler problem, looking for a pattern, writing an
equation, performing an experiment, guessing at a solution and then check
our result.

3.3.3.3 Carry Out the Plan

Once we devised a plan, we must carry it out. Careful execution of the


plan is important and a record of the process and results obtained shall be in
place. If the first plan is not successful, we can always use another plan and
start all over.

3.3.3.4 Review the Solution

Once we have found a solution, we check the solution and make sure
that the solution is consistent with the facts of the problem. We must interpret
the solution in the context of the problem and figure out whether there are
generalizations of the solution that could apply to other problems.

Page 14 of 20
Example 10. Apply Polya’s strategy to solve the following problem.

Determine the digit 100 places to the right of the decimal point in the
4
decimal representation 27.

Solution.
Understand What are the given  The number
4
7
the Problem information?
 100 places to the right of the decimal point

What is asked?  100𝑡ℎ number to the right of the decimal point in the decimal
4
representation of .
7

What is/are the


property/ies of the  One of the numbers in the set {0,1,2,3,4,5,6,7,8,9}
final answer?
Devise a Plan What steps are 1.
4
Express in decimal form.
7
needed to solve the
2. Make a list of the digits to the right of the decimal
problem?
notation.
3. Look for a pattern.

Carry Out the What are the 1.


4
The number in decimal notation is given by
27
Plan results after we
0.148148148 … which has a repeating and non-
take the steps in
terminating pattern.
solving the
2.
problem?
Location Digit Location Digit Location Digit
1𝑠𝑡 1 2𝑛𝑑 4 3𝑟𝑑 8
4𝑡ℎ 1 5𝑡ℎ 4 6𝑡ℎ 8
7𝑡ℎ 1 8𝑡ℎ 4 9𝑡ℎ 8
10𝑡ℎ 1 11𝑡ℎ 4 12𝑡ℎ 8
13𝑡ℎ 1 14𝑡ℎ 4 15𝑡ℎ 8

3. Since the decimal representation repeats 148 over and


over, we know that the digit located 100 places to the
right of the decimal point is either 1, 4, or 8.

Only in column 3 is each of the decimal digit locations


evenly divisible by 3. From this pattern we can tell that
the 99𝑡ℎ decimal digit must be 8. Since a 1 always
follows an 8 in the pattern, the 100𝑡ℎ decimal digit must
be a 1.

Review the Are we sure that  As an alternative solution, we see that the above table
Solution our answer is illustrates additional patterns. For instance, if each of the
correct? location numbers in column 1 is divided by 3, a remainder 1
is produced. If each of the location numbers in column 2 is
divided by 3, a remainder of 2 is produced. Thus, we can
find the decimal digit in any location by dividing the location
number by 3 and examining the remainder. Thus, to find
4
the digit in the 100th decimal place of , we merely divide
27
100 by 3 and examine the remainder, which is 1. Thus, the
digit 100 places to the right of the decimal point is a 1.

Page 15 of 20
Example 11. Apply Polya’s strategy to solve the following problem.

A paper fan and a visor together cost ₱100.00. The visor costs ₱ 90.00
more than the paper fan. What are the individual costs of the visor and the
fan?

Solution.
Understand What are the given  The price of a paper fan and a visor together is ₱100.00.
the Problem information?  The visor costs ₱90.00 more than the paper fan.

What is asked?  Cost of the visor


 Cost of the fan

What is/are the  Visor’s price must be higher than the price of the paper
property/ies of the fan.
final answer?  Both prices must be less than ₱100.00.
 Unit of cost is in ₱.

Devise a What steps are 1. Represent the cost of visor and paper fan using a variable.
Plan needed to solve 2. Translate the statement to mathematical equation.
the problem? 3. Solve for the cost of the visor and the paper fan.
Carry Out What are the 1. We let ℎ be the cost of the paper fan. Since the visor costs
the Plan results after we ₱90.00 more than the visor, we represent its cost by ℎ +
take the steps in 90.
solving the 2. The cost of paper fan and visor altogether is ₱100.00. This
problem? can be expressed ℎ + ℎ + 90 = 100.
3. Solving this equation, we have
ℎ + ℎ + 90 = 100
2ℎ + 90 = 100
2ℎ = 100 − 90
2ℎ = 10
ℎ=5
The cost of the paper fan is ₱5.00 and the cost of the visor
is
₱ 5.00 + ₱ 90.00 = ₱ 95.00.

Review the Are we sure that  Checking the sum of the costs we get ₱ 5.00 + ₱ 95.00 =
Solution our answer is ₱ 100.00
correct?  The difference of the costs of the visor and the paper fan
is ₱95.00 − ₱5.00 = ₱90.00.

Page 16 of 20
Learning Activity 6

Direction. Use Polya’s four steps to problem solving in solving the


following problems.

A true or false quiz contains 10 questions. In how many ways can a


student answer the questions if the student answers two of the
questions with false and the rest with true?

Understand the
problem

Devise a plan

Carry out the


plan

Review the
solution

Determine the units digit of 4320 .

Understand the
problem

Devise a plan

Carry out the


plan

Review the
solution

Page 17 of 20
3.4 Recommended learning materials and resources

 Can You Solve “Einstein’s Riddle”? – Dan Van der Vieren


https://www.youtube.com/watch?v=1rDVz_Fb6HQ

 Recreational game: Tower of Hanoi


https://www.mathsisfun.com/games/towerofhanoi.html

 Recreational game: KenKen puzzle


 http://www.kenkenpuzzle.com/#

3.5 Flexible Teaching Learning Modality (FTLM)

Remote (asynchronous)

 module, exercises, problems sets, powerpoint lessons

3.6 Assessment Task

A. Determine whether the given argument is an example of inductive


reasoning (IR) or deductive reasoning(DR). Write your answer on the blank
after each item.

1. Emma enjoyed reading the novel Under the Dome by


Stephen King, so she will enjoy reading his next novel.

2. All pentagons have exactly five sides. Figure 𝐴 is a


pentagon. Therefore, Figure 𝐴 has exactly five sides.

3. Cats do not eat tomatoes. Tigger is a cat. Therefore, Tigger


does not eat tomatoes.

4. A number is a neat number if the sum of the cubes of its


digits equals the number. Therefore, 153 is a neat
number.

5. The Mengals have won five games in a row. Therefore, the


Mengals will win their next game.

Page 18 of 20
B. Write the answer to the questions on the space provided after each
item.

1. What is the next number in the list?


80, 70, 61, 53, 46, 40, ?

2. What is the next number in the list?


2, 7, −3, 2, −8, −3, −13, −8, −18, ?

3. Find a pair of numbers to show that the given


statement is false.
“If the sum of two counting numbers is an even counting
number, then the product of the two counting numbers
is an even counting number.”

4. Find a pair of numbers to show that the given


statement is false.
“If the product of two counting numbers is an even
counting number, then both of the counting numbers are
even counting numbers.”

5. Use deductive reasoning to give the result of


the given procedure:

Pick a number. Multiply the number by 6 and add 8.


Divide the sum by 2, subtract twice the original number,
and subtract 4.

C. Problem Solving. Solve the following problems using Polya’s four steps
to problem solving or your alternative(systematic) process.

1. What is the 𝑛th- term formula for the figure below?

2. The bacteria in a petri dish grow in a manner such that each day
the number of bacteria doubles. On what day will the number of
bacteria be half of the number present on the 12th day?

Page 19 of 20
3. The number of ducks and pigs in a field total 35. The total
number of legs among them is 98. Assuming each duck has
exactly two legs and each pig has exactly four legs, determine
how many ducks and how many pigs are in the field?

4. A room measures 12 feet by 15 feet. How many 3-foot by 3-foot


squares of carpet are needed to cover the floor of this room?

5. There was a jar of chocolate chip cookies on the table. James


and Monica were very hungry because they hadn’t had anything
to eat since breakfast, so they ate half the cookies. Then Victor
came along and noticed the cookies. He ate a third of what was
left in the jar. Sharon, who was waiting around nearby, decided
to take a fourth of the cookies left in the jar. Then Tifanny came
rushing up and took one cookie to munch on in her class. When
Valerie looked at the cookie jar, she saw there were two cookies
left. How many cookies were in the jar to begin with?

Your answers in Problem Solving will be graded according to the given


standards/basis for grading:

Score Criteria
0 You did not make any attempt to solve the problem.
1 You identified what is asked in the problem.
You identified what is asked in the problem as well as other needed
2
information.
You identified what is asked in the problem, the needed information, and
3
used a correct process. However, you did not obtain the correct answer.
You were able to completely solve the problem using a correct process.
4
However, you did not give a justification that your solution is indeed correct.
You were able to completely solve the problem using a correct process and
5
with a review of your solution.

3.7 References

Books:

Aufmann, et. al. 2013. Mathematical Excursions. Third Edition. Cengage


Learning, USA

Hammack, Richard. 2013 Book of Proof. Department of Mathematics and


Applied Mathematics. Virginia Commonwealth University

Stewart, I. 2008. Professor Stewarts’s Cabinet of Mathematical Curiosities.


Basic Books. USA

Page 20 of 20

You might also like